At the bookstore, best-sellers are normally $19.95 each. During the store-wide 20% off sale, how much would it cost you, before tax, to buy two best-sellers?
F. $3.99
G. $7.98
H. $15 96
J. $31.92
K. $39.90

Answers

Answer 1

Step-by-step explanation:

20% off means you pay 80 % of the original price for two books

80% * ( 2 x 19.95 ) = .8 ( 39.90) = $ 31.92


Related Questions

Tristan is going to invest in an account paying an interest rate of 6.2% compounded monthly. How much would Tristan need to invest, to the nearest cent, for the value of the account to reach $18,100 in 12 years?

Answers

Answer:

Step-by-step explanation:

We can use the compound interest formula to solve this problem:

A = P(1 + r/n)^(nt)

Where:

A = Final amount (which is given as $18,100)

P = Principal amount (the amount Tristan needs to invest)

r = Annual interest rate (6.2%)

n = Number of times interest is compounded per year (12)

t = Time in years (12)

Substituting the values in the formula:

18,100 = P(1 + 0.062/12)^(12*12)

Simplifying the right-hand side:

18,100 = P(1.00516666667)^144

Dividing both sides by (1.00516666667)^144:

P = 18,100 / (1.00516666667)^144

P = $8,167.75 (rounded to the nearest cent)

Therefore, Tristan needs to invest $8,167.75, to the nearest cent, for the value of the account to reach $18,100 in 12 years.

Can someone please help me with this?

Answers

The triangles within kite with corresponding angles and sides are similar, so options A, B, D, and E are true statements while options C and F are wrong.

What are similar triangles

Similar triangles are two triangles that have the same shape, but not necessarily the same size. This means that corresponding angles of the two triangles are equal, and corresponding sides are in proportion.

The line FH bisects the kite so:

triangle ∆FGK is similar to ∆FJK, ∆FGK ≅ ∆FJK

angle m∠GKH corresponds to m∠JKH, m∠GKH ≅ m∠JKH

angle m∠GFH corresponds to m∠JFH, m∠GFH ≅ m∠JFH

triangle ∆GKH is similar to ∆JKH, ∆GKH ≅ ∆JKH.

The line FG is not shown to be same with KG, likewise lines FG and JK, so lines FG ≅ KG and

FG ≅ JK are not true statements.

Therefore, options A, B, D, and E are true statements for the triangles within the kite with corresponding angles and sides that are similar, while options C and F are wrong.

Know more about similar triangles here:https://brainly.com/question/14285697

#SPJ1

Find m ∠ R . Use the Picture

Answers

m∠R is therefore 76 degrees as a triangle's total sides equal 180 degrees.

what is angle ?

The degree of rotation between two lines or two planes around a central point is measured by an angle. Typically, it is expressed in radians or degrees. Angles are used in many mathematical and scientific uses, including trigonometry, physics, and engineering, where they are crucial in determining the shape and characteristics of geometric figures. Angles come in four different varieties: acute (less than 90 degrees), right (exactly 90 degrees), oblique (more than 90 degrees), and straight (exactly 180 degrees).

given

Because a triangle's total sides equal 180 degrees, we have:

R, S, and T add up to 180.

Inputting the numbers provided yields:

m∠R + 72 + 32 = 180

Simplifying the equation:

m∠R = 76

m∠R is therefore 76 degrees as a triangle's total sides equal 180 degrees.

To know more about angles visit:

https://brainly.com/question/14569348

#SPJ1

A beef rancher randomly sampled 42 cattle from her large herd to obtain a 95% confidence interval to estimate the mean weight of the cows in the herd. The interval obtained was (1010, 1321). If the rancher had used a 90% confidence interval instead, the interval would have been
(A) wider and would have more precision than the original estimate
(B) narrower and would have more precision than the original estimate
(C) wider and would have the same precision as the original estimate
(D) narrower and would have less precision than the original estimate
(E) wider and would have less precision than the original estimate

Answers

(E)The interval would have been wider and would have less precision than the original  .

A beef rancher randomly sampled 42 cattle from her large herd to obtain a 95% confidence interval to estimate the mean weight of the cows in the herd. The interval obtained was (1010, 1321). If the rancher had used a 90% confidence interval instead, the interval would have been narrower and would have less precision than the original estimate.

If the sample size is constant, then we can say that as the level of confidence is increased, the width of the interval will also increase. This is because a higher level of confidence will require a larger interval to include the population mean. Similarly, if the level of confidence is decreased, then the width of the interval will also decrease as a smaller interval is needed to include the population mean.

This directly implies that if the confidence interval is wider, then it has more precision and vice versa. Hence, Option (E) is the correct answer.

To learn more about confidence interval refer :

https://brainly.com/question/24131141

#SPJ11

which sampling approach was used in the following statement?kane randomly sampled 250 nurses from urban areas and 250 from rural areas from a list of licensed nurses in wisconsin to study their attitudes toward evidence-based practice.

Answers

The sampling approach that was used in the statement "Kane randomly sampled 250 nurses from urban areas and 250 from rural areas from a list of licensed nurses in Wisconsin to study their attitudes toward evidence-based practice" is Stratified random sampling.

What is Stratified random sampling?

Stratified random sampling is a method of sampling that is based on dividing the population into subgroups called strata. Stratified random sampling is a statistical sampling method that involves the division of the population into subgroups or strata, and a sample is then drawn from each stratum in proportion to the size of the stratum. It's a sampling method that ensures the representation of all population strata in the sample, making it more effective than simple random sampling.

Stratified random sampling is used when there are variations in the population that are likely to influence the outcome of the study. The stratified random sampling method is used to ensure that these differences are reflected in the sample. In this way, the results of the study are more representative of the entire population than they would be if a simple random sample were used.

Learn more about Stratified random sampling here: https://brainly.com/question/20544692.

#SPJ11

Write down two factors of 24 that are primenumber

Answers

the prime factors of 24 are 2 and 3, which combine to give the unique prime factorization of 24 as 2^3 × 3.

There are no factors of 24 that are prime numbers. A factor of a number is a whole number that divides that number without leaving a remainder. Prime numbers, on the other hand, are numbers that are divisible only by 1 and themselves, and cannot be expressed as the product of any other numbers.

The prime factors of 24 are 2, 2, and 3. We can factorize 24 as 2 × 2 × 2 × 3 or 2^3 × 3. Here, 2 and 3 are both prime numbers, but they are not factors of 24 in isolation. They are only prime factors of 24 when combined in the manner shown.

This fact highlights an important concept in number theory: the uniqueness of prime factorization. Every composite number can be expressed as a unique product of prime numbers. This fundamental theorem of arithmetic is crucial in many areas of mathematics, including cryptography, where it is used to secure communications and protect sensitive information.

In summary, there are no factors of 24 that are prime numbers. However, the prime factors of 24 are 2 and 3, which combine to give the unique prime factorization of 24 as 2^3 × 3.

To know more about prime factorization click here:

brainly.com/question/29763746

#SPJ4

You are on the observation deck of the Empire State building looking at the Chrysler building when you turn 145° clockwise you see the Statue of Liberty you know that the Chrysler building in the Empire State building or about 0.6 miles apart and that the Chrysler building in the Statue of Liberty are about 5.6 miles apart estimate the distance between the Empire State building in the Statue of Liberty round your answer to the nearest 10th of a mile

Answers

I think it might be 6 but I might be wrong

If the original quantity is 225 and the quality of 320 what is the percentage increase?

Answers

Step-by-step explanation:

The change fro 225 to 320 is 95

95 is what percentage of 225?

95/225 x 100% = 42.2% increase

What are the x coordinate and the y coordinate of the center of mass for the uniform plate shown in Figure below if L=5.0 cm?

Answers

From the  X coordinate is -0.45 centimeter and Y coordinate is -2.0 cm of the center of mass for the uniform plate.

According to the Question:

Since the plate is uniform, we can divide it into three rectangles, each with a mass proportional to its area and with its barycenter at its geometric center. We will refer to the large piece of 35cm x 10cm; it occupies 63.6% of the total surface and its barycenter is at

(x₁, y₁) = (−5.0cm,−2.5cm).

Now,

The top 20cm× 5cm piece has 18.2 % of the total area; its center of mass is at (x₂, y₂) = (10cm,12.5cm).  

Now,

The bottom 10cm× 10cm piece (section 3) also has 18.2 % of the total area; its center of mass is at (x₃, y₃)​ = (5cm,−15cm).

(a) The x coordinate of the center of mass for the plate is

   [tex]X_{com} = (0.636)X_1 + (0.182)X_2 + (0.182)X_3[/tex]

            = -0.45 cm

(b) The y coordinate of the center of mass for the plate is

    [tex]Y_{com} = (0.636)Y_1 + (0.182)Y_2 + (0.182)Y_3[/tex]

             = -2.0 cm.

Learn more about Coordinate:

https://brainly.com/question/16634867

#SPJ4

When a homeowner has a 25-year variable-rate mortgage loan, the monthly payment R is a function of the amount of the loan A and the current interest rate i (as a percent); that is, R = f(A). Interpret each of the following. (a) R140,000, 7) - 776.89 For a loan of $140,000 at 7% interest, the monthly payment is $776.89. For a loan of $140,000 at 7.7689% interest, 700 monthly payments would be required to pay off the loan. For a loan of $140,000 at 7% interest, 776.89 monthly payments would be required to pay off the loan. For a loan of $140,000 at 7.7689% interest, the monthly payment is $700.

Answers

The monthly payment required to pay off a loan of $140,000 at 7% interest would be $776.89 is the correct statement(A).

The statement given is describing a function that relates the monthly payment R of a 25-year variable-rate mortgage loan to the loan amount A and the current interest rate i.

The given values are R = $776.89 and A = $140,000, with an interest rate of 7%. This means that the monthly payment required to pay off a loan of $140,000 at 7% interest would be $776.89.

However, the other statements are incorrect interpretations. For instance, the statement "For a loan of $140,000 at 7.7689% interest, 700 monthly payments would be required to pay off the loan" is incorrect.

This is because the number of payments required to pay off a loan depends not only on the loan amount and interest rate, but also on the term of the loan.

Similarly, the statement "For a loan of $140,000 at 7% interest, 776.89 monthly payments would be required to pay off the loan" is also incorrect, as the number of payments required would be determined by the term of the loan.

Finally, the statement "For a loan of $140,000 at 7.7689% interest, the monthly payment is $700" is also incorrect. This is because, for the given loan amount and interest rate, the monthly payment required would be $776.89, as calculated above.

For more questions like Interest click the link below:

https://brainly.com/question/13324776

#SPJ11

1. Use least-squares regression to fit a straight line to x 6 7 11 15 17 21 23 29 29 37 39 y 29 21 29 14 21 15 7 7 13 0 3 Along with the slope and the intercept, compute the standard error of the estimate and the correlation coefficient. Please write a detailed solution with explanation

Answers

The least-squares regression analysis can provide important information about the relationship between two variables, including the slope and intercept of the fitted line, the standard error of the estimate, and the correlation coefficient.

What are the steps to use least-squares regression?

To use least-squares regression to fit a straight line to the given data points, follow these steps:

Calculate the means of x and y values:
  x_mean = (6 + 7 + 11 + 15 + 17 + 21 + 23 + 29 + 29 + 37 + 39) / 11 = 21.18
  y_mean = (29 + 21 + 29 + 14 + 21 + 15 + 7 + 7 + 13 + 0 + 3) / 11 = 14.55

Calculate the sums of squared differences for x and y:
  Σ(x-x_mean)^2 = Σ(y-y_mean)² = Σ(x-x_mean)(y-y_mean) = 0
  (Compute these sums using the given x and y values)

Calculate the slope (b) of the fitted straight line:
  b = Σ(x-x_mean)(y-y_mean) / Σ(x-x_mean)²

Calculate the intercept (a) of the fitted straight line:
  a = y_mean - b * x_mean
Calculate the standard error of the estimate (SEE):
  SEE = sqrt(Σ(y-y_pred)² / (n-2))
  (y_pred represents the predicted y values using the fitted straight line equation)

Calculate the correlation coefficient (r):
  r = Σ(x-x_mean)(y-y_mean) / sqrt(Σ(x-x_mean)² x  Σ(y-y_mean)²

Once you have followed these steps using the given x and y values, you will have the least-squares regression line, along with the slope and intercept, the standard error of the estimate, and the correlation coefficient.

Learn more about least-squares regression

brainly.com/question/30044885

#SPJ11

Consider the initial value problem D'=13_), zo = [ 2] y(0) = a. Find the eigenvalue 1, an eigenvector v1, and a generalized eigenvector v2 for the coefficient matrix of this linear system. - l= , Vj = -- b. Find the most general real-valued solution to the linear system of differential equations. Use t as the independent variable in your answers. y(t) = C1 + C2 c. Solve the original initial value problem. yı(t) = ya(t) =

Answers

The solution to the initial value problem is yı(t) = ya(t) = [a e^(13t) - (e^(13t) - 1)] (1, 0).

Given information:Consider the initial value problem D' = 13_, zo = [ 2] y(0) = a.To find the eigenvalue 1, an eigenvector v1, and a generalized eigenvector v2 for the coefficient matrix of this linear system.Step-by-step explanation:The given differential equation isD' = 13_Y = [ 2] y(0) = a.The coefficient matrix A for this system is:A = [13]The eigenvalues λ1, λ2 are obtained by solving the characteristic equation:det(A - λI) = 0where I is the 2x2 identity matrixdet(A - λI) = (13 - λ)(-λ) - 2(0) = λ(λ - 13)This equation has two roots:λ1 = 0, λ2 = 13.The corresponding eigenvectors v1 and v2 are obtained by solving the equations:(A - λ1I) v1 = 0, (A - λ2I) v2 = 0. For λ1 = 0, we have(A - λ1I) v1 = (13 0) (v1[1])= (0 0) (v1[2])which implies that v1[1] = 0 and v1[2] is free. Therefore, v1 = (0, 1). For λ2 = 13, we have(A - λ2I) v2 = (0 0) (v2[1])= (0 -13) (v2[2])which implies that v2[1] is free and v2[2] = 0. Therefore, v2 = (1, 0).The generalized eigenvector v3 for λ1 = 0 is obtained by solving the equation:(A - λ1I) v3 = v2For this equation, we have(13 0) (v3[1])= (0 0) (v3[2])which implies that v3[1] is free and v3[2] = 0. Therefore, v3 = (1, 0).b. Find the most general real-valued solution to the linear system of differential equations. Use t as the independent variable in your answers.The general solution of the system is given byy(t) = c1 e^(0 t) (0, 1) + c2 e^(13t) (1, 0) + c3 (t e^(0 t) (0, 1) + e^(0 t) (1, 0))Therefore, the most general real-valued solution to the linear system of differential equations is given byy(t) = c1(0, 1) + c2 e^(13t)(1, 0) + c3 (t(0, 1) + (1, 0))c. Solve the original initial value problem.To solve the given initial value problem, we need to determine the coefficients c1, c2, and c3 that satisfy the initial conditions:y(0) = a = c1(0, 1) + c2(1, 0) + c3(0, 1) + (1, 0)c1 = 0, c2 = a - 1, and c3 = 0Therefore, the solution to the initial value problem is given byy(t) = (a - 1) e^(13t) (1, 0) + (1, 0) = [a e^(13t) - (e^(13t) - 1)] (1, 0) = y1(t)The solution to the initial value problem is yı(t) = ya(t) = [a e^(13t) - (e^(13t) - 1)] (1, 0).

Learn more about Initial

brainly.com/question/13243199

#SPJ11

Enter the correct answer in the box
Function is shown on the graph in function is the parent quadratic function what is the equation of transtormed function in terms of ?

Answers

In quadratic functiοn -

          g(x)                                    h(x)                                     d(x)

vertical shift dοwn 3     reflectiοn acrοss the x-axis   vertical shift dοwn 3

hοrizοntal shift left 3     vertical strecht οf 3               hοrizοntal shift right 3

What is the οperatiοn οf a quadratic functiοn?

The expressiοn f(x) = ax² + bx + c, where a, b, and c are all integers with a nοt equal tο zerο, denοtes a quadratic functiοn. The shape that a quadratic functiοn's graph takes οn is called a parabοla.

In spite οf the fact that parabοlas might have οpenings that are upward οr dοwnward, varying "widths" οr "steepnesses," they all have the same basic "U" shape.

Quadratic parent: f(x)=x²

The graph is a parabοla with vertex V=(0,0) at the οrigin and οpens up.

When x=1→f(1)=1²→f(1)=1

1) g(x)

The graph οpens up, then there is nοt a reflectiοn acrοss the x-axis.

The vertex is at the pοint (-3,-3): 3 units tο the left and 3 units dοwn οf the vertex οf the parent funtiοn.

When x is 1 unit tο the right frοm the vertex g(x)=1

Then the transfοrmatiοns were applied tο the cuadratic parent functiοn are:

1.1) vertical shift dοwn 3.

1.2) hοrizοntal shift left 3.

2) h(x)

The graph οpens dοwn, then there is a reflectiοn acrοss the x-axis.

The vertex is at the οrigin (0,0).

When x is 1 unit tο the right frοm the vertex h(x)=-3

Then the transfοrmatiοns were applied tο the quadratic parent functiοn are:

2.1) reflectiοn acrοss the x-axis.

2.2) vertical strecht οf 3.

3) d(x)

The graph οpens up, then there is nοt a reflectiοn acrοss the x-axis.

The vertex is at the pοint (3,-3): 3 units tο the right and 3 units dοwn οf the vertex οf the parent funtiοn.

When x is 1 unit tο the right frοm the vertex d(x)=1

Then the transfοrmatiοns were applied tο the cuadratic parent functiοn are:

3.1) vertical shift dοwn 3.

3.2) hοrizοntal shift right 3.

Learn more about quadratic function

brainly.com/question/30929439

#SPJ1

The complete question is -

The given graphs show functions which have been transformed from the quadratic parent, f(x) = x2. Determine which transformations were applied to the quadratic parent function to result in each graph.

I need help I need to show my work please help

Answers

Answer:

The 2nd equation is false.

Step-by-step explanation:

40 ≠ 58, so we can say the 2nd equation is false.

Use a double integral to find the area of the region D.
The x y-coordinate plane is given. There is a curve that encloses a region.
The curve, labeled r = √theta, starts at the origin, goes counterclockwise and away from the origin, and ends on the positive x-axis.
The region, labeled D, is the area enclosed by the curve and the positive x-axis.

Answers

Geographic area D has an area of /8 square units.

What is the formula for the area?

Area is a phrase used to describe how much room a 2D form or surface occupies. Area is measured in square units, such as cm2 or m2. The area of a form is computed by dividing its length by its breadth.

What really is area in plain English?

Area is the entire amount of space occupied by a plain (2-D) area or an object's form. The area of a planar figure is the area that its perimeter encloses. The quantity of unit square that cover a closed figure's surface is its area.

0 = √θ

θ = 0

Similarly, the curve intersects the [tex]x-axis[/tex]  again when [tex]x = r cos[/tex] θ = √θ cos θ = 0, which implies θ = π/2.

Thus, the limits of integration for θ are [tex]0[/tex] and π/2. For r, the curve is given by [tex]r =[/tex] √θ, so the limits of integration are [tex]0[/tex] and √(π/2).

Evaluating this integral gives:

∫(θ=0 to π/2) [[tex]1/2 r^2[/tex]] (r=0 to √(π/2)) dθ

= ∫(θ=0 to π/2) [[tex]1/2[/tex] (π/2)] dθ

= π/8

Therefore, the area of region D is π/8 square units.

To know more about area visit:

https://brainly.com/question/27683633

#SPJ1

The volume of the right cone below is 5677 units³. Find the value of x.​

Answers

According to the given information value of X =16.06

What is the volume of right cone?

The volume of a cone defines the space or the capacity of the cone. A cone is a three-dimensional geometric shape having a circular base that tapers from a flat base to a point called apex or vertex. A cone is formed by a set of line segments, half-lines or lines connecting a common point, the apex, to all the points on a base that is in a plane that does not contain the apex.

A cone can be seen as a set of non-congruent circular disks that are stacked on one another such that the ratio of the radius of adjacent disks remains constant.

According to the given information:

Cone volume formula

A cone is a solid that has a circular base and a single vertex. To calculate its volume, you need to multiply the base area (area of a circle: π × r²) by height and by 1/3:

volume = (1/3) × π × r² × h

5677 = (1/3) x 22/7 x r² x 21

x = 16.06

Therefore, according to the given information value of X =16.06

To know more about click:

brainly.com/question/10625069

#SPJ1

ABCD is a quadrilateral A) Calculate the value of x. B) When ABCD is drawn to scale, would the lines AD and BC be parallel or not? You must justify your answer without using a scale drawing

Answers

A) The value of  x = 45 degrees

B) Lines AD and BC are not parallel when ABCD is drawn to scale.

To solve this problem, we can use the fact that the sum of the angles in a quadrilateral is 360 degrees.

A) angle A + angle B + angle C + angle D = 360

2x + 90 + x + 3x = 360

6x + 90 = 360

6x = 270

x = 45

Therefore, x = 45 degrees.

B) To determine if lines AD and BC are parallel, we can look at the opposite angles of the quadrilateral. If they are supplementary (add up to 180 degrees), then the lines are parallel.

angle A + angle C = 2x + x = 3x = 135 degrees

angle B + angle D = 90 + 3x = 90 + 135 = 225 degrees

Since angle A + angle C and angle B + angle D do not add up to 180 degrees, the opposite angles are not supplementary, and therefore, lines AD and BC are not parallel.

Learn more about quadrilateral here

brainly.com/question/9550663

#SPJ4

The given question is incomplete, the complete question is:

ABCD is a quadrilateral A) Calculate the value of x. B) When ABCD is drawn to scale, would the lines AD and BC be parallel or not?

Stacy, Eileen, and Michelle are on a basketball team. Stacy scored 15% of the points, Eileen
scored 10% and Michelle scored 13% of their team's points. If their team had a total
points, how many points did the remaining players on the team score?

Answers

The points scored by the remaining players on the team is 31 points.

What is percentage?

A component or fraction of a whole can be expressed as a percentage by using a number out of 100. Mathematicians frequently use comparison to represent changes in quantities over time or to compare two numbers. For instance, if a student receives an 80 percent on a test, they properly answered 80 of the 100 questions.

For performing a range of mathematical operations, such as determining the percentage rise or decrease of a quantity or computing interest rates on loans or investments, percentages can be expressed as fractions or decimals.

From the given percentages the total percent is:

Total percentage = 15% + 10% + 13% = 38%

Now, from 100 the remaining percentage is: 100% - 38% = 62%

For the remaining players the score is given as:

Remaining points = 50 * (62/100) = 31

Hence, the points scored by the remaining players on the team is 31 points.

Learn more about percentage here:

https://brainly.com/question/29763752

#SPJ1

Need help with question 4 please

Answers

Answer:

Question 4

a) 0.667

b) 0.833

c) 0.833

d) 0

Question 5

a) 32 yards

b) $312

Step-by-step explanation:

Probability question 4

The best way to solve these probability questions is to enumerate and count the total set of outcomes and then enumerate and count the specific set of outcomes they are looking for in each part .

Since the experiment consists of rolling a fair single 6-sided die  we can enumerate the set of  possible outcomes as
{1, 2, 3, 4, 5, 6}. This is also referred to in probability as sample space.

There are 6 possible outcomes

The probability of getting any one number is 1/6 whether it be a 1, a 2 or 3, 4, 5 or 6.

In general P(a specific outcome) = Number of ways of getting that outcome ÷ total number of all possible outcomes

part a)
Rolling an even==>  Set of outcomes  = {2, 4, 6}
Rolling a 3 => {3}

Rolling a 3 or even = {2, 3, 4, 6}; 4 elements
P(even or 3 ) = 4/6 = 2/3 = 0.667

Part b)
Number > 2 ==> { 3, 4, 5, 6}
Even: {2, 4, 6}

Even or greater than 2 = {2, 3, 4, 5, 6}; 5 elements  - we do not duplicate

P(Even or greater than 2 ) = 5/6 = 0.833

part c)
Odd number: {1, 3, 5}
Number less than 6: {1, 2, 3, 4, 5}
Combining the two we get: {1, 2, 3, 4, 5} for a total of 5 outcomes

P(odd or less-than 6 ) = 5/6 = 0.833

part(d)
Rolling a 2 and a 4
You cannot roll a 2 and a 4 in one toss so the probability is 0

Question 5
I think you have overly complicated the working of this problem. There is no need to use right triangle, hypotenuse etc

If you have a rectangle with length = L and width = W, the perimeter is just

P = 2( L + W)

Here we can choose any dimension for L and the other for W
Let's choose L = 11 (greater number)
W = 5

Perimeter P = 2 (11 + 5) = 2(16) = 32 yards

b) It costs $3.25 per foot for the fencing..
Careful!
The  only tricky part here is the units. It is given in $ per square foot whereas the dimensions of the plot are given in yards and the perimeter is also in yards. Watch out for these tricks in other questions

Convert perimeter from yards to feet

32 yards = 32 x 3 = 96 feet  (there are 3 feet to a yard)

Cost of 96 feet of fencing at $3.25 per foot
= 96 x 3.25 = $312


What are the expectations for the following $1 bet on a U.S. roulette wheel? (Round each answer to the nearest cent.)
0 & 00 split = $

Answers

The expectation for a $1 bet on a U.S. roulette wheel for a 0 & 00 split is -$2. This means that you are expected to lose $2 for every $1 you bet.

The expected payout for a $1 bet on the 0 & 00 split in a U.S. roulette wheel is $0.45.

On a U.S. roulette wheel, there are 38 possible outcomes, consisting of the numbers 1 through 36, 0, and 00.

The 0 and 00 numbers are green, while all other numbers are either red or black.

Placing a $1 bet on the 0 & 00 split means that the bettor is betting that either the 0 or the 00 number will hit. If either of these numbers hits, the payout is 17 to 1, meaning the bettor will receive $17 for every $1 bet.

To calculate the expected payout for a $1 bet on the 0 & 00 split, we multiply the probability of winning (1/38) by the payout ($17), resulting in an expected payout of $0.447.

Therefore, the expected payout for a $1 bet on the 0 & 00 split is:

(1/38) × 17 × $1 = $0.447

Rounding this to the nearest cent, the expected payout for a $1 bet on the 0 & 00 split is $0.45.

Hence, the answer is $0.45.

To learn more about expected payout:

https://brainly.com/question/24855677

#SPJ11

Determine whether the planes are parallel, perpendicular, or neither. If neither, find the angle between them. x + 4 y − 3 z = 1 , − 3 x + 6 y + 7 z = 0

Answers

The angle between the two planes is approximately 64.63°.

The two planes x + 4 y − 3 z = 1 and − 3 x + 6 y + 7 z = 0 are neither parallel nor perpendicular. The angle between them can be determined by the following formula:

[tex]\theta = \cos^{-1}\left(\frac{A\cdot B}{\left\lvert A\right\rvert\left\lvert B\right\rvert}\right)[/tex]
where A and B are the normal vectors of the two planes, and \theta is the angle between them.

A normal vector for the plane x + 4 y − 3 z = 1 can be calculated as:

A = \begin{bmatrix} 1 \\ 4 \\ -3 \end{bmatrix}

A normal vector for the plane − 3 x + 6 y + 7 z = 0 can be calculated as:

B = \begin{bmatrix} -3 \\ 6 \\ 7 \end{bmatrix}

The angle between the two planes can be found by plugging the vectors into the formula above:

[tex]\theta

= \cos^{-1}\left(\frac{A\cdot B}{\left\lvert A\right\rvert\left\lvert B\right\rvert}\right)

= \cos^{-1}\left(\frac{(-3)\cdot(-3) + 6 \cdot 4 + 7 \cdot (-3)}{\sqrt{1 + 16 + 9}\sqrt{9 + 36 + 49}}\right)

= \cos^{-1}\left(\frac{-37}{\sqrt{26}\sqrt{94}}\right)\approx 64.63^\circ[/tex]

for such more questions on angle between vectors

https://brainly.com/question/31062862

#SPJ11

In a class of students, the following data table summarizes how many students passed
a test and complete the homework due the day of the test. What is the probability that
a student completed the homework given that they failed the test?
Completed the homework
Did not complete the homework
Passed the test Failed the test
8
2
3
сл

Answers

The probability that a student completed the homework given that they failed the test is 3/8.

Describe Probability?

Probability is a branch of mathematics that deals with the study of randomness and uncertainty in events. It involves the measurement and analysis of the likelihood or chance of an event occurring. Probability is expressed as a number between 0 and 1, with 0 indicating an impossible event and 1 indicating a certain event.

The concept of probability is used in many fields, such as statistics, physics, finance, and engineering, to make predictions and decisions based on incomplete information. It provides a framework for analyzing situations in which multiple outcomes are possible and assigns a quantitative value to the likelihood of each outcome.

To find the probability that a student completed the homework given that they failed the test, we need to use conditional probability.

Let A be the event that a student completed the homework and B be the event that a student failed the test.

We want to find P(A | B), the probability that a student completed the homework given that they failed the test.

Using the data from the table, we can see that the total number of students who failed the test is 8 (3 who completed the homework and 5 who did not).

Therefore, P(B) = 8/18 = 4/9 (the probability that a student failed the test)

Out of the 8 students who passed the test, 3 also completed the homework, so P(A and not B) = 3/18 = 1/6.

So, P(A | B) = P(A and B) / P(B) = (1/6) / (4/9) = 3/8.

Therefore, the probability that a student completed the homework given that they failed the test is 3/8.

To know more about outcome visit:

https://brainly.com/question/29815200

#SPJ1

A triangle has two sides of length 3 and 16. What is the largest possible whole-number length for the third side

Answers

The largest possible whole-number length for the third side is 18, which satisfies all three inequalities.

What is inequality theorem?

The triangle inequality theorem explains the relationship between the three sides of a triangle. This theorem states that for any triangle, the sum of the lengths of the first two sides is always larger than the length of the third side.

According to question:

Let x be the length of the third side. By the triangle inequality, we have:

3 + 16 > x and 16 + x > 3 and 3 + x > 16

Simplifying, we get:

19 > x and x > 13 and x < 19

The largest possible whole-number length for the third side is 18, which satisfies all three inequalities.

To know more about Inequality visit:

brainly.com/question/30228778

#SPJ1

In an aquarium, the ratio of dolphins to pufferfish is 2: 3 and the ratio of pufferfish to starfish is 4: 7. There are 8 dolphins in the aquarium. How many starfish are there?​

Answers

There ratio are [tex]28[/tex] starfish inside an aquarium, with a dolphin to pufferfish ratio of 1.

What else are ratio in simple simple words?

The ratio may be defined as the number of ways you can represent one item as a percentage of another. Only when the two quantities in a ratio share the same unit can they be compared. Ratios are employed to compare two items.

How do you compute ratio?

Ratios contrast two figures by typically dividing them. A/B would be your formula if you were comparing each data point (A) to that other data point (B). This indicates that you are multiplying info A by info B. If A is five and B is ten, for instance, your ratio will indeed be 5/10.

Dolphins to pufferfish to starfish: [tex]2:3:7[/tex]

So for every [tex]2[/tex] dolphins, there are [tex]7[/tex] starfish. We know there are 8 dolphins, so we can set up a proportion:

[tex]2[/tex] dolphins / [tex]7[/tex] starfish = 8 dolphins / x starfish

where [tex]x[/tex] is the number of starfish.

Solving for x, we get:

[tex]x[/tex] starfish = (7 starfish * [tex]8[/tex] dolphins) / [tex]2[/tex] dolphins

[tex]x[/tex] starfish = [tex]28[/tex] starfish

Therefore, there are [tex]28[/tex] starfish in the aquarium.

To know more about ratio visit:

https://brainly.com/question/867293

#SPJ1

In a random sample of 200 school district residents, 94 stated they are in favor of starting the school day 15 minutes later each day. Calculate a 90% confidence interval for the true proportion of district residents who are in favor of starting the day later

Answers

The 90% confidence interval for the proportion of district residents in favor of starting the school day 15 minutes later is (0.392, 0.548). The true proportion is estimated to lie within this interval with 90% confidence.

To calculate the 90% confidence interval for the true proportion of district residents who are in favor of starting the school day 15 minutes later, we can use the following formula:

CI = p ± z*(√(p*(1-p)/n))

where:

CI: confidence interval

p: proportion of residents in favor of starting the day later

z: z- score based on the confidence level (90% in this case)

n: sample size

First, we need to calculate the sample proportion:

p = 94/200 = 0.47

Next, we need to find the z- score corresponding to the 90% confidence level. Since we want a two-tailed test, we need to find the z- score that cuts off 5% of the area in each tail of the standard normal distribution. Using a z-table, we find that the z- score is 1.645.

Substituting the values into the formula, we get:

CI = 0.47 ± 1.645*(√(0.47*(1-0.47)/200))

Simplifying this expression gives:

CI = 0.47 ± 0.078

Therefore, the 90% confidence interval for the true proportion of district residents who are in favor of starting the school day 15 minutes later is (0.392, 0.548). We can be 90% confident that the true proportion lies within this interval.

Learn more about the confidence interval here: brainly.com/question/24131141

#SPJ4

Make up a sequences that have (a) 3,3,3,3,... as its second differences. (b) 1, 2,3,4,5,... as its third differences (c) 1, 2, 4,8,16,... as its 100th differences.

Answers

The nth term of the sequence is 2^n.

(a) 3, 3, 3, 3, ... is a sequence that has 0 for both its first and second differences. That is, every term in the sequence is the same.(b) The sequence is the series of natural numbers. It has 0 for its first and second differences, and 6 for its third differences. The nth term of the sequence is n.(c) The sequence has 0 for its first 99 differences and 100! for its 100th difference. The nth term of the sequence is 2^n.

Learn more about Sequence

brainly.com/question/1606838

#SPJ11

Rob says -3(x-2) is equivalent to -3x-6. What was his error?

A) heshould have written -3x-2 which would have given him the right answer.
B) hedid not multiply 3 with x to get 3x, and instead put -3x.
C) he had no error.
D) he did not multiply -3 with -2 to get +6 instead of -6

Answers

Answer:

D) he did not multiply -3 with -2 to get +6 instead of -6

Step-by-step explanation:

-3(x - 2)

-3x + 6

Compare the two

-3x + 6 ≠ -3x - 6

They are not equal! Looking at the options, D is the answer.

Answer:

The correct answer is D) he did not multiply -3 with -2 to get +6 instead of -6.

To expand -3(x-2), we need to distribute -3 to both terms inside the parentheses, which gives us:

-3(x-2) = -3x + (-3(-2)) = -3x + 6

Therefore, the correct equivalent expression is -3x + 6, not -3x - 6 as Rob wrote. He made a mistake in multiplying -3 and -2, which should have given him +6 instead of -6.

(please mark my answer as brainliest)

Find the derivative of the function f(x), below. It may be to your advantage to simplify before differentiating. f(x)=ln(14-e^-2x). f'(x)

Answers


To find the derivative of the function f(x), we need to take the derivative of the natural logarithm (ln) of the function. We can do this by using the chain rule, which states that the derivative of the composition of two functions is equal to the derivative of the outer function times the derivative of the inner function.

The derivative of the outer function (ln) is 1/f(x), and the derivative of the inner function (14 - e-2x) is -2e-2x. So the derivative of f(x) is:

f'(x) = 1/f(x) × (-2e-2x)

f'(x) = 1/(ln(14 - e-2x)) × (-2e-2x)

f'(x) = -2e-2x/(14 - e-2x)

To learn more about “derivatives” refer to the : https://brainly.com/question/12047216

#SPJ11

a manager recorded the number of gallons of ice cream sold for the past six periods. he asked you to choose a forecasting model to predict the demand for gallons of ice cream in period 7. you consider applying a two-period moving average model and a two-period weighted moving average model with weights of 0.6 and 0.4. a) which model is better for this data set (hint: show all your work including forecasts for each period and calculations using measures of forecast accuracy)? (9 points)

Answers

The two-period moving average model and the two-period weighted moving average model are both common forecasting methods used to predict future demand. and we understand that the model with the lower MAD and MSE values will have the most accurate forecast.

To determine which model is better for this particular data set, we need to compare the accuracy of each model. To do this, we will calculate the Mean Absolute Deviation (MAD) and the Mean Squared Error (MSE) for each model.
For the two-period moving average model, we can calculate the forecast for period 7 by taking the average of p5 and 6:

Period 7 forecast = (Gallons in Period 5 + Gallons in Period 6)/2

For the two-period weighted moving average model, we can calculate the forecast for period 7 by using the weights of 0.6 and 0.4:

Period 7 forecast = (0.6 x Gallons in Period 5) + (0.4 x Gallons in Period 6)

We can then compare the accuracy of each model by calculating the MAD and MSE. To calculate MAD, we need to subtract the actual demand in each period from the forecasted demand and take the absolute value:

MAD = |Actual demand – Forecasted demand|

To calculate MSE, we need to square the differences between the actual demand and the forecasted demand:

MSE = (Actual demand – Forecasted demand)^2

After calculating the MAD and MSE for each model, we can compare the results to determine which model is better for this data set. The model with the lower MAD and MSE values will have the most accurate forecast.

For more such questions on forecast

https://brainly.com/question/24730207

#SPJ11

Find the total number of outcomes for picking a day of the week and a month of the year. A 84 , B 19 , C 60 , D 210

Answers

Option A is the correct option for determining the total number of possible outcomes when selecting a day of the week and a month of the year, which is 84.

To find the total number of outcomes for picking a day of the week and a month of the year, you need to multiply the number of options for each category. There are 7 days in a week, so there are 7 options for picking a day. There are 12 months in a year, so there are 12 options for picking a month. To find the total number of outcomes, you multiply the number of options for each category: 7 x 12 = 84. Therefore, the total number of outcomes for picking a day of the week and a month of the year is 84. Option A is the correct answer.

Learn more about probability here: brainly.com/question/30034780

#SPJ4

Other Questions
One of the best ways to develop a sustainable competitive advantage is to differentiate your product or make your product unique in a way that is valuable to your customers. T/F At the end of the first year of operations, the balance sheet of Huntington Beach Co. Industries had the following balances: Accounts Receivable, $5,000; Accounts Payable, $6,000; Inventory, $3,000; and Unexpired Insurance, $2,000. The corporation reported net income of $79,000 for the year, including depreciation expense of $5,000, and uses the indirect method of computing net cash flow from operating activities. Based on this information, net cash flow from operating activities is a. $78,000 b. $77,000 c. $70,000 d. $82,000 Determine the number of factors of the given whole number waves that have traveled out of their area of origin and have become uniform and symmetrical are known as . It's final exam time and you're finding yourself extremely stressed out. If we examined your blood, which of the following would we be likely to find?Select one:a. Negative feedback to the adrenal cortex by adrenocorticotropic hormone (ACTH)b. Decreased secretion of corticotropin-releasing hormone (CRH) by the hypothalamusc. Increased secretion of corticotropin from the anterior pituitaryd. Increased secretion of corticotropin from the hypothalamus You are studying a population of sea jellies. You take DNA samples and sequence a specific locus with two alleles for each individual, and get the following genotype frequencies:KK=0.16,Kk=0.59, and kk=0.25a) Is this population in Hardy-Weinberg equilibrium? Defend your answer b) Could these allele frequencies happen in a real population? Why or why not? The ____________________ perform photosynthesis, have chlorophyll, and produce oxygen as a by-product.a. cyanobacteriab. archaebacteriac. proteobacteriad. ribosomes what affects the patterns of prevailing winds, and how does it do so? for the fiscal year ending december 31, previous year and the current year, justin co. has net sales of $1,000,000 and $2,000,000; average gross receivables of $100,000 and $300,000; and allowance for uncollectible accounts receivable of $30,000 and $50,000, respectively. if the accounts receivable turnover and the ratio of allowance for uncollectible accounts receivable to gross accounts receivable are calculated, which of the following best represents the conclusions to be drawn?A. Accounts receivable turnovers in day are10.0and6.6respectively. Examine allowance for possible overstatement of the allowance. B. Accounts receivable turnovers in days are36.5and54.8respectively. Examine allowance for possible understatement of the allowance, C. Accounts receivable turnovers in days are14.3and8.0respectively. Examine allowance for possible overstatement of the allowance. D. Accounts receivable turnovers in days are14.3and8.0respectively. Examine allowance for possible understatement of the allowance. MOM+DAD=FAST DAD is a multiple of 83 cryptarithm How would this program be affected if the on button block were deleted fromthe code?on menu button pressedshow long text HellobottomA. Text would be shown, but it would not be the text "Hello."OB. No text would be shown when the "Menu" button was pressed.OC. The text would be shown without the player pressing the "Menu"button.D. Nothing would change, because the on button block does not havea specific purpose in a game. Co(NH3)6 has a maximum absorbance in the UV-Vis of 475 nm and appears yellow-orange. If we assume this is due to the crystal field splitting, what is A in J/molecule?What is A in kJ/mole? In a hostel, 50 students have food enough for 54 days. How many students should be added in the hostel so that the food is enough for only 45 days ? Which of the following characteristics are relevant for determining which federal court has jurisdiction over a case?- location where the case was filed(Most federal cases start at the district court level, of which there are 94 in the country. Appeals from these courts go to one of 12 circuit courts of appeal, and which court is chosen is a matter of which state the case starts in.)- subject matter of the case(Some federal courts have jurisdictions that are based on the subject matter of the case, rather than geography. For example, the U.S. Court of International Trade hears all trade and customs issues.)- the parties in the case(The Supreme Court is the court of original jurisdiction for cases between two states, whereas a lower federal court will hear a case between an individual and the government.) which consumes more energy, a 1.2 kw hair dryer used for 10 min or a 10 w night light left on for 24 hr what information does the first and third digit of the VESPR number provide When she turned 23, Rita purchased 165 shares of Stock A for $15 per share; 64 shares of Stock B for $23 per share; 34 shares of Stock C for $76 per share; and 125 shares of Stock D for $24 per share. At age 34, Rita sold Stock A for $1980 with annual dividends of $85 per year, Stock B for $1928, Stock C for $2718, and Stock D for $3398. Which stock provided Reese with greatest return on her investment?a. Stock Ab. Stock Bc. Stock Cd. Stock D Although messages may have a primary and a secondary audience, the writer needs to profile only the primary audience to determine the best presentation of the message. True or false after a musical concert that you enjoyed without distraction, a companion complains that she heard little of the music due to frequent shuffling and throat-clearing in the audience. the difference in your experience and hers is related to 11- An object of mass 0.2 kg moves in a circular path. If it makes 3/4 revolution in 0.3 s and its displacement is 6 m, calculate: a) The radius of the circular path ?